Differental Equation homework (w/ calc 2 intergrals)

  • Thread starter Thread starter mandymandy
  • Start date Start date
  • Tags Tags
    Calc 2 Homework
Click For Summary
The discussion focuses on solving a differential equation related to population dynamics, specifically the carrying capacity N. The equation dP/dt = KP(N-P) describes how population P changes over time, with conditions for growth and decline based on its relation to N. The user is attempting to integrate the equation using partial fractions, as suggested by their teacher, to find P(t). A hint is provided to review the integration technique involving ln and partial fractions for the integral ∫dP/P(N-P). The conversation emphasizes the importance of understanding these mathematical concepts to solve the problem effectively.
mandymandy
Messages
3
Reaction score
0

Homework Statement


Carrying capacity N of a population

Assume: small population, dp/dt is proportional to P

if 0<P<N then p(t) increases
if P>N then P(t) decreases
equilibrium means dp/dt = 0


Homework Equations


dP/dT = KP(N-P)

Solve for P(t)


The Attempt at a Solution


∫dP/P(N-P) = ∫kdt ?

My teacher said we'd be using ln(...) and 1/[P(N-P)] = A/P + B/(N-P)


Please help! Thanks. :)
 
Physics news on Phys.org
mandymandy said:

Homework Statement


Carrying capacity N of a population

Assume: small population, dp/dt is proportional to P

if 0<P<N then p(t) increases
if P>N then P(t) decreases
equilibrium means dp/dt = 0


Homework Equations


dP/dT = KP(N-P)

Solve for P(t)


The Attempt at a Solution


∫dP/P(N-P) = ∫kdt ?

My teacher said we'd be using ln(...) and 1/[P(N-P)] = A/P + B/(N-P)


Please help! Thanks. :)

You are on the right track. Perhaps you need to review partial fractions to work the integral$$
\int \frac{1}{P(N-P)}\, dP$$Your teacher gave you a pretty good hint there.
 
I can solve it from here I don't know why I was so confused. :) thanks
 
Question: A clock's minute hand has length 4 and its hour hand has length 3. What is the distance between the tips at the moment when it is increasing most rapidly?(Putnam Exam Question) Answer: Making assumption that both the hands moves at constant angular velocities, the answer is ## \sqrt{7} .## But don't you think this assumption is somewhat doubtful and wrong?

Similar threads

  • · Replies 7 ·
Replies
7
Views
6K
  • · Replies 4 ·
Replies
4
Views
2K
  • · Replies 1 ·
Replies
1
Views
2K
  • · Replies 5 ·
Replies
5
Views
2K
  • · Replies 1 ·
Replies
1
Views
2K
  • · Replies 5 ·
Replies
5
Views
4K
  • · Replies 7 ·
Replies
7
Views
3K
  • · Replies 7 ·
Replies
7
Views
2K
  • · Replies 2 ·
Replies
2
Views
812
Replies
7
Views
2K